IASbaba Prelims 60 Days Plan, Rapid Revision Series (RaRe)
Archives
Hello Friends
The 60 Days Rapid Revision (RaRe) Series is IASbaba’s Flagship Initiative recommended by Toppers and loved by the aspirants’ community every year.
It is the most comprehensive program which will help you complete the syllabus, revise and practice tests on a daily basis. The Programme on a daily basis includes
Daily Prelims MCQs from Static (Monday – Saturday)
- Daily Static Quiz will cover all the topics of static subjects – Polity, History, Geography, Economics, Environment and Science and technology.
- 20 questions will be posted daily and these questions are framed from the topics mentioned in the schedule.
- It will ensure timely and streamlined revision of your static subjects.
Daily Current Affairs MCQs (Monday – Saturday)
- Daily 5 Current Affairs questions, based on sources like ‘The Hindu’, ‘Indian Express’ and ‘PIB’, would be published from Monday to Saturday according to the schedule.
Daily CSAT Quiz (Monday – Friday)
- CSAT has been an Achilles heel for many aspirants.
- Daily 5 CSAT Questions will be published.
Note – Daily Test of 20 static questions, 10 current affairs, and 5 CSAT questions. (35 Prelims Questions) in QUIZ FORMAT will be updated on a daily basis.
To Know More about 60 Days Rapid Revision (RaRe) Series – CLICK HERE
60 Day Rapid Revision (RaRe) Series Schedule – CLICK HERE
Important Note
- Comment your Scores in the Comment Section. This will keep you accountable, responsible and sincere in days to come.
- It will help us come out with the Cut-Off on a Daily Basis.
- Let us know if you enjoyed today’s test 🙂
- You can post your comments in the given format
- (1) Your Score
- (2) Matrix Meter
- (3) New Learning from the Test
Test-summary
0 of 35 questions completed
Questions:
- 1
- 2
- 3
- 4
- 5
- 6
- 7
- 8
- 9
- 10
- 11
- 12
- 13
- 14
- 15
- 16
- 17
- 18
- 19
- 20
- 21
- 22
- 23
- 24
- 25
- 26
- 27
- 28
- 29
- 30
- 31
- 32
- 33
- 34
- 35
Information
The following Test is based on the syllabus of 60 Days Plan-2023 for UPSC IAS Prelims 2022.
To view Solutions, follow these instructions:
- Click on – ‘Start Test’ button
- Solve Questions
- Click on ‘Test Summary’ button
- Click on ‘Finish Test’ button
- Now click on ‘View Questions’ button – here you will see solutions and links.
You have already completed the test before. Hence you can not start it again.
Test is loading...
You must sign in or sign up to start the test.
You have to finish following test, to start this test:
Results
0 of 35 questions answered correctly
Your time:
Time has elapsed
You have scored 0 points out of 0 points, (0)
Average score |
|
Your score |
|
Categories
- Not categorized 0%
Pos. | Name | Entered on | Points | Result |
---|---|---|---|---|
Table is loading | ||||
No data available | ||||
- 1
- 2
- 3
- 4
- 5
- 6
- 7
- 8
- 9
- 10
- 11
- 12
- 13
- 14
- 15
- 16
- 17
- 18
- 19
- 20
- 21
- 22
- 23
- 24
- 25
- 26
- 27
- 28
- 29
- 30
- 31
- 32
- 33
- 34
- 35
- Answered
- Review
-
Question 1 of 35
1. Question
With reference to the Ecologists and their research’, consider the following pairs:
- Antonie van Leeuwenhoek : First introduced the concept of food chains
- Herbert Spencer : Coined the term ‘survival of the fittest’
- Tansley : Coined the term ‘ecology’
How many of the above given pairs is/are correctly matched?
Correct
Solution (b)
Ecology
Ecology can be defined as the scientific exploration of living organisms and their interactions with each other and their environment. The term “ecology” was coined by Ernst Haeckel, a German biologist, in 1869. It originates from the Greek words “Oikos,” meaning home or estate, and “logos,” meaning study. The main focus of ecology is to understand the relationships between organisms and the abiotic (non-living) and biotic (living) components of their environment. It investigates how organisms are influenced by their surroundings and how they utilize environmental resources, including energy flow and mineral cycling.
Timeline of Ecologists:
- Antonie van Leeuwenhoek (1632–1723): First introduced the concept of food chains. (Hence pair 1 is correctly matched)
- Carl Linnaeus (1707–1778): Renowned naturalist and founder of scientific study on natural economics.
- Alexander Humboldt (1769–1859): First described the latitudinal biodiversity gradient toward the tropics.
- Charles Darwin (1809–1882): Proposed the idea of natural selection and conducted ecological studies on soils.
- Herbert Spencer (1820–1903): Coined the term “survival of the fittest.” (Hence pair 2 is correctly matched)
- Karl Möbius (1825–1908): Developed the concept of ecological community or living community.
- Ernst Haeckel (1834–1919): Coined the term “ecology” and popularized the study of ecology in relation to evolution. (Hence pair 3 is incorrectly matched)
- Vladimir Vernadsky (1869–1939): Introduced the concept of the biosphere.
- Henry C. Cowles (1869–1939): Pioneered studies on ecological succession.
- Tansley (1871–1955): Coined the term “ecosystem” and was a prominent researcher.
- Henry Gleason (1882–1975): Founder of the individualistic concept of ecology and a quantitative theorist.
- Charles S. Elton (1900–1991): Known as the Father of Animal Ecology, he developed the concepts of food chains and niches.
- Evelyn Hutchinson (1903–1991): Elaborated on the niche concept in ecology.
Incorrect
Solution (b)
Ecology
Ecology can be defined as the scientific exploration of living organisms and their interactions with each other and their environment. The term “ecology” was coined by Ernst Haeckel, a German biologist, in 1869. It originates from the Greek words “Oikos,” meaning home or estate, and “logos,” meaning study. The main focus of ecology is to understand the relationships between organisms and the abiotic (non-living) and biotic (living) components of their environment. It investigates how organisms are influenced by their surroundings and how they utilize environmental resources, including energy flow and mineral cycling.
Timeline of Ecologists:
- Antonie van Leeuwenhoek (1632–1723): First introduced the concept of food chains. (Hence pair 1 is correctly matched)
- Carl Linnaeus (1707–1778): Renowned naturalist and founder of scientific study on natural economics.
- Alexander Humboldt (1769–1859): First described the latitudinal biodiversity gradient toward the tropics.
- Charles Darwin (1809–1882): Proposed the idea of natural selection and conducted ecological studies on soils.
- Herbert Spencer (1820–1903): Coined the term “survival of the fittest.” (Hence pair 2 is correctly matched)
- Karl Möbius (1825–1908): Developed the concept of ecological community or living community.
- Ernst Haeckel (1834–1919): Coined the term “ecology” and popularized the study of ecology in relation to evolution. (Hence pair 3 is incorrectly matched)
- Vladimir Vernadsky (1869–1939): Introduced the concept of the biosphere.
- Henry C. Cowles (1869–1939): Pioneered studies on ecological succession.
- Tansley (1871–1955): Coined the term “ecosystem” and was a prominent researcher.
- Henry Gleason (1882–1975): Founder of the individualistic concept of ecology and a quantitative theorist.
- Charles S. Elton (1900–1991): Known as the Father of Animal Ecology, he developed the concepts of food chains and niches.
- Evelyn Hutchinson (1903–1991): Elaborated on the niche concept in ecology.
-
Question 2 of 35
2. Question
Which of the following sequence correctly orders the levels of organizations in ecology from the smallest to the greatest?
Correct
Solution (a)
Levels of Organisations in Ecology
- Levels of the organization are natural systems that are frequently described by part-whole relationships, with higher ones being made up of lower levels. The individual, population, community, ecosystem, biome, and biosphere are the levels of organization in ecology.
- Ecosystems can be researched on a local or large scale. From the smallest to the greatest, the levels of organisation are detailed below:
Organism → Population → Community → Ecosystem → Biome → Biosphere (Hence option (a) is correct)
Incorrect
Solution (a)
Levels of Organisations in Ecology
- Levels of the organization are natural systems that are frequently described by part-whole relationships, with higher ones being made up of lower levels. The individual, population, community, ecosystem, biome, and biosphere are the levels of organization in ecology.
- Ecosystems can be researched on a local or large scale. From the smallest to the greatest, the levels of organisation are detailed below:
Organism → Population → Community → Ecosystem → Biome → Biosphere (Hence option (a) is correct)
-
Question 3 of 35
3. Question
With reference to the difference between Environment and Ecology, consider the following statements:
- Ecology specifically focuses on the interactions between organisms and their surroundings while Environment examines the interaction of physical, chemical, and biological components.
- The ecology encompasses all aspects of a living thing’s existence, while environment has a narrower scope centered on the study of organisms and their environment
- Environmental elements encompass temperature, water, light, air, soil, and nutrients, whereas ecology focuses on the study of different ecosystems and how animals depend on each other for survival
How many of the above statements are correct?
Correct
Solution (b)
Environment:
- Refers to the interaction of physical, chemical, and biological components.
- Encompasses the circumstances in which living things exist, ranging from small-scale settings to larger ecosystems.
- Examines both internal and external factors that influence the environment, such as pollution, global warming, and deforestation.
- Includes environmental elements like temperature, water, light, air, soil, and nutrients.
Ecology:
- The study of how organisms interact with their surroundings.
- Focuses on understanding life processes, adaptability, distribution, and biodiversity.
- Explores the intricate relationships and interactions among organisms and their environment.
- Considers factors like population size, organism dispersion, species diversity, and competition.
- Involves studying various ecosystems and how animals depend on one another to survive.
Differences between Environment and Ecology:
- Environment examines the interaction of physical, chemical, and biological components, while ecology specifically focuses on the interactions between organisms and their surroundings. (Hence statement 1 is correct)
- The environment encompasses all aspects of a living thing’s existence, while ecology has a narrower scope centered on the study of organisms and their environment. (Hence statement 2 is incorrect)
- Environmental concerns include issues like pollution, global warming, and deforestation, while ecological considerations involve population size, organism dispersion, diversity, and competition.
- Environmental elements encompass temperature, water, light, air, soil, and nutrients, whereas ecology focuses on the study of different ecosystems and how animals depend on each other for survival. (Hence statement 3 is correct)
- The term “eco” is often associated with environmentally friendly practices that do not harm specific species or substances.
Incorrect
Solution (b)
Environment:
- Refers to the interaction of physical, chemical, and biological components.
- Encompasses the circumstances in which living things exist, ranging from small-scale settings to larger ecosystems.
- Examines both internal and external factors that influence the environment, such as pollution, global warming, and deforestation.
- Includes environmental elements like temperature, water, light, air, soil, and nutrients.
Ecology:
- The study of how organisms interact with their surroundings.
- Focuses on understanding life processes, adaptability, distribution, and biodiversity.
- Explores the intricate relationships and interactions among organisms and their environment.
- Considers factors like population size, organism dispersion, species diversity, and competition.
- Involves studying various ecosystems and how animals depend on one another to survive.
Differences between Environment and Ecology:
- Environment examines the interaction of physical, chemical, and biological components, while ecology specifically focuses on the interactions between organisms and their surroundings. (Hence statement 1 is correct)
- The environment encompasses all aspects of a living thing’s existence, while ecology has a narrower scope centered on the study of organisms and their environment. (Hence statement 2 is incorrect)
- Environmental concerns include issues like pollution, global warming, and deforestation, while ecological considerations involve population size, organism dispersion, diversity, and competition.
- Environmental elements encompass temperature, water, light, air, soil, and nutrients, whereas ecology focuses on the study of different ecosystems and how animals depend on each other for survival. (Hence statement 3 is correct)
- The term “eco” is often associated with environmentally friendly practices that do not harm specific species or substances.
-
Question 4 of 35
4. Question
What is the significance of understanding the niche of a species in conservation efforts?
Correct
Solution (c)
Niche:
Understanding the niche of a species provides valuable insights into its habitat requirements, population dynamics, and ecological interactions. This knowledge is vital for identifying key habitat areas, managing populations, and developing effective conservation strategies to safeguard species and preserve biodiversity. (Hence option (c) is correct answer)
- A niche refers to the unique functional role or place of a species in an ecosystem.
- It encompasses all the biological, physical, and chemical factors that a species requires to survive, stay healthy, and reproduce.
- No two species have identical niches; each species has its own unique niche.
- Understanding the niche of a species is crucial for conservation efforts.
- Conservation of a species in its native habitat requires knowledge about its niche requirements and ensuring that all those requirements are fulfilled.
There are different types of niches:
- Habitat niche: Refers to the specific location where a species lives.
- Food niche: Involves the species’ diet, including what it eats or decomposes, and its interactions with other species for food resources.
- Reproductive niche: Describes the species’ reproductive strategies, including how and when it reproduces.
- Physical niche: Encompasses physical and environmental factors such as temperature, land slope, humidity, and other specific requirements for the species.
Incorrect
Solution (c)
Niche:
Understanding the niche of a species provides valuable insights into its habitat requirements, population dynamics, and ecological interactions. This knowledge is vital for identifying key habitat areas, managing populations, and developing effective conservation strategies to safeguard species and preserve biodiversity. (Hence option (c) is correct answer)
- A niche refers to the unique functional role or place of a species in an ecosystem.
- It encompasses all the biological, physical, and chemical factors that a species requires to survive, stay healthy, and reproduce.
- No two species have identical niches; each species has its own unique niche.
- Understanding the niche of a species is crucial for conservation efforts.
- Conservation of a species in its native habitat requires knowledge about its niche requirements and ensuring that all those requirements are fulfilled.
There are different types of niches:
- Habitat niche: Refers to the specific location where a species lives.
- Food niche: Involves the species’ diet, including what it eats or decomposes, and its interactions with other species for food resources.
- Reproductive niche: Describes the species’ reproductive strategies, including how and when it reproduces.
- Physical niche: Encompasses physical and environmental factors such as temperature, land slope, humidity, and other specific requirements for the species.
-
Question 5 of 35
5. Question
Which process in the Phosphorus Cycle involves the breakdown of organic materials into inorganic forms of phosphorus by bacteria?
Correct
Solution (d)
Phosphorus Cycle:
- Phosphorus undergoes a cycle, moving through rocks, water, soil, sediments, and organisms.
- Rain and weathering gradually release phosphate ions and other minerals from rocks.
- Inorganic phosphate is then distributed throughout the soil and water.
- Plants absorb inorganic phosphate from the soil, while animals consume plants for their phosphorus needs.
- Once inside plants or animals, phosphate is incorporated into organic molecules like DNA.
- Upon the death and decomposition of plants or animals, organic phosphate is released back into the soil.
- Bacteria play a role in the process of mineralization, breaking down organic materials into inorganic forms of phosphorus and making them available to plants in the soil. (Hence option (d) is correct)
- Phosphorus from the soil can find its way into streams and eventually reach the oceans.
- Over time, it can become absorbed into sediments in these locations.
Incorrect
Solution (d)
Phosphorus Cycle:
- Phosphorus undergoes a cycle, moving through rocks, water, soil, sediments, and organisms.
- Rain and weathering gradually release phosphate ions and other minerals from rocks.
- Inorganic phosphate is then distributed throughout the soil and water.
- Plants absorb inorganic phosphate from the soil, while animals consume plants for their phosphorus needs.
- Once inside plants or animals, phosphate is incorporated into organic molecules like DNA.
- Upon the death and decomposition of plants or animals, organic phosphate is released back into the soil.
- Bacteria play a role in the process of mineralization, breaking down organic materials into inorganic forms of phosphorus and making them available to plants in the soil. (Hence option (d) is correct)
- Phosphorus from the soil can find its way into streams and eventually reach the oceans.
- Over time, it can become absorbed into sediments in these locations.
-
Question 6 of 35
6. Question
Consider the following Cycles:
- Water (Hydrologic) Cycle
- The Carbon Cycle
- The Nitrogen Cycle
- Sulphur Cycle
Which of the above given cycles are considered as Gaseous Cycles?
Correct
Solution (a)
Gaseous Cycles:
Water (Hydrologic) Cycle:
- The water (hydrologic) cycle refers to the continuous circulation of water in the Earth-atmosphere system, driven by solar energy.
- Various reservoirs such as the atmosphere, oceans, lakes, rivers, soils, glaciers, snowfields, and groundwater store water in our world.
- Processes like evaporation, transpiration, condensation, precipitation, deposition, runoff, infiltration, and groundwater flow facilitate the movement of water between reservoirs.
- The hydrologic cycle also involves the continuous exchange of water between the land surface, oceans, subsoil, and organisms.
- It begins with the evaporation of water from the ocean’s surface.
The Carbon Cycle:
- Carbon, mainly in the form of carbon dioxide (CO2), exists in the atmosphere.
- The carbon cycle involves the continual exchange of carbon between the atmosphere and organisms.
- Through processes like photosynthesis, carbon is transferred from the atmosphere to green plants and subsequently to other organisms.
- It returns to the atmosphere through respiration and the decomposition of dead organic matter. It is considered a relatively short-term cycle.
The Nitrogen Cycle:
- Nitrogen is a fundamental component of living tissues and plays a crucial role in proteins, accounting for approximately 16 percent of protein weight.
- The nitrogen cycle consists of three main phases: nitrogen fixation, nitrification, and denitrification.
- It involves the atmosphere, hydrosphere, and lithosphere.
- Nitrogen fixation is the anaerobic process in which atmospheric nitrogen (N2) is converted into NH3 (ammonia) by nitrogen-fixing bacteria.
- Nitrification is a two-step process where ammonium ions (NH4+) are first reduced to NO2 and then further oxidized to produce NO3. Soil bacteria are involved in both steps.
Denitrification is the conversion of nitrates (NO3) back into nitrogen gas. Denitrifying bacteria perform a similar function as nitrogen-fixing bacteria.
Incorrect
Solution (a)
Gaseous Cycles:
Water (Hydrologic) Cycle:
- The water (hydrologic) cycle refers to the continuous circulation of water in the Earth-atmosphere system, driven by solar energy.
- Various reservoirs such as the atmosphere, oceans, lakes, rivers, soils, glaciers, snowfields, and groundwater store water in our world.
- Processes like evaporation, transpiration, condensation, precipitation, deposition, runoff, infiltration, and groundwater flow facilitate the movement of water between reservoirs.
- The hydrologic cycle also involves the continuous exchange of water between the land surface, oceans, subsoil, and organisms.
- It begins with the evaporation of water from the ocean’s surface.
The Carbon Cycle:
- Carbon, mainly in the form of carbon dioxide (CO2), exists in the atmosphere.
- The carbon cycle involves the continual exchange of carbon between the atmosphere and organisms.
- Through processes like photosynthesis, carbon is transferred from the atmosphere to green plants and subsequently to other organisms.
- It returns to the atmosphere through respiration and the decomposition of dead organic matter. It is considered a relatively short-term cycle.
The Nitrogen Cycle:
- Nitrogen is a fundamental component of living tissues and plays a crucial role in proteins, accounting for approximately 16 percent of protein weight.
- The nitrogen cycle consists of three main phases: nitrogen fixation, nitrification, and denitrification.
- It involves the atmosphere, hydrosphere, and lithosphere.
- Nitrogen fixation is the anaerobic process in which atmospheric nitrogen (N2) is converted into NH3 (ammonia) by nitrogen-fixing bacteria.
- Nitrification is a two-step process where ammonium ions (NH4+) are first reduced to NO2 and then further oxidized to produce NO3. Soil bacteria are involved in both steps.
Denitrification is the conversion of nitrates (NO3) back into nitrogen gas. Denitrifying bacteria perform a similar function as nitrogen-fixing bacteria.
-
Question 7 of 35
7. Question
With reference to the Ecological succession, consider the following statements:
- Ecological succession is a common phenomenon observed in all communities, and it is closely connected to the process of evolution
- Secondary succession progresses at a slower pace in comparison to primary succession
- Pioneer species are the first organisms to colonize an area during secondary succession, playing a crucial role in the establishment of subsequent communities.
How many of the above statements are correct?
Correct
Solution (a)
Ecological Succession:
- Ecological succession refers to the gradual and progressive growth of species in a specific area in response to changing environmental conditions.
- It aims to establish equilibrium in the ecological system through the development of a climax community, which represents a stable and diverse community of organisms.
- The process involves constant changes in the number and types of species within a given area, leading to the formation of different communities known as seral communities.
- Ecological succession is a common phenomenon observed in all communities, and it is closely connected to the process of evolution. (Hence statement 1 is correct)
- There are two main types of ecological succession: primary succession and secondary succession.
- Primary succession occurs in lifeless areas where there is no soil or in barren lands that cannot support life. The process starts from scratch, with rocks being broken down and soil formation being a crucial step.
- Secondary succession occurs when an existing ecosystem is disrupted or destroyed, allowing new species to colonize the area. This type of succession is faster because there is already a foundation of soil and life present. (Hence statement 2 is incorrect)
- Pioneer species are the first organisms to colonize an area during primary succession, playing a crucial role in the establishment of subsequent communities. (Hence statement 3 is incorrect)
- The stages of ecological succession involve a series of changes in species composition, vegetation structure, and ecological dynamics, leading to the development of a climax community.
- Ecological succession is essential for the initiation and persistence of life on Earth, and it plays a vital role in shaping ecosystems and their biodiversity.
Incorrect
Solution (a)
Ecological Succession:
- Ecological succession refers to the gradual and progressive growth of species in a specific area in response to changing environmental conditions.
- It aims to establish equilibrium in the ecological system through the development of a climax community, which represents a stable and diverse community of organisms.
- The process involves constant changes in the number and types of species within a given area, leading to the formation of different communities known as seral communities.
- Ecological succession is a common phenomenon observed in all communities, and it is closely connected to the process of evolution. (Hence statement 1 is correct)
- There are two main types of ecological succession: primary succession and secondary succession.
- Primary succession occurs in lifeless areas where there is no soil or in barren lands that cannot support life. The process starts from scratch, with rocks being broken down and soil formation being a crucial step.
- Secondary succession occurs when an existing ecosystem is disrupted or destroyed, allowing new species to colonize the area. This type of succession is faster because there is already a foundation of soil and life present. (Hence statement 2 is incorrect)
- Pioneer species are the first organisms to colonize an area during primary succession, playing a crucial role in the establishment of subsequent communities. (Hence statement 3 is incorrect)
- The stages of ecological succession involve a series of changes in species composition, vegetation structure, and ecological dynamics, leading to the development of a climax community.
- Ecological succession is essential for the initiation and persistence of life on Earth, and it plays a vital role in shaping ecosystems and their biodiversity.
-
Question 8 of 35
8. Question
With reference to the ‘Ecological Adaptation’, consider the following statements:
- Phenotypic plasticity is the capacity of organisms to alter their physical characteristics in accordance with the surrounding environment
- Sciophytes are plants that exhibit a preference for shade and conditions with low light levels
- Reproduction in humans serves as an instance of an ecological adaptation
How many of the above statements are correct?
Correct
Solution (b)
Ecological Adaptations
- The topic of organisms and their environment explores how living beings develop special characteristics to thrive in specific environmental conditions. These adaptations can be morphological, physiological, or behavioral in nature, and they arise through the process of natural selection, either from existing variations or through gradual selection of small adaptive traits.
- The main purpose of these adaptations is to enhance an individual’s ability to obtain food and space for survival, find opportunities for reproduction, and successfully raise offspring.
- Reproduction in humans is a biological process that ensures the continuation of the species. While reproduction is a fundamental aspect of an organism’s life cycle, it is not specifically considered an ecological adaptation. (Hence statement 3 is incorrect)
- Ecological adaptations typically refer to traits or behaviors that directly aid an organism in its interaction with the environment, such as camouflage in animals to blend with their surroundings, drought tolerance in plants to survive arid conditions, or migration in birds to find more favorable habitats during different seasons
Phenotypic Plasticity and Ecotypes
- Phenotype refers to the observable traits resulting from the interaction between an organism’s genetic makeup (genotype) and its surrounding environment. Within a local habitat, variations in environmental conditions lead to diverse phenotypes, a phenomenon known as phenotypic plasticity. (Hence statement 1 is correct)
- These variations give rise to ecotypes, which are genetically adapted local populations of species that exhibit morphological and physiological differences. Despite these genetic variations, ecotypes can interbreed. Ecotypes are commonly observed in plants and sessile animals.
Strategies of Adaptations in Plants
- Plants possess unique characteristics that enable them to tolerate a wide range of environmental conditions, such as light regimes, dryness, high temperatures, water-saturation, and saline environments.
- They have evolved specialized structures in their flowers to ensure pollination by insects or other animals. Additionally, plants have developed various mechanisms to cope with stressful environmental conditions.
Adaptation to Light Regime
Based on their adaptation to different light intensities, plant communities can be categorized into two groups:
- Sciophytes: These are shade-loving plants that exhibit lower levels of photosynthetic, respiratory, and metabolic activities. They typically grow under the dense canopy of forests and include ferns and herbaceous plants. (Hence statement 2 is correct)
- Heliophytes: These plants are adapted to high-intensity light and have higher temperature thresholds for photosynthesis. They also exhibit higher rates of respiration. Examples of heliophytes include shrubs and trees.
Incorrect
Solution (b)
Ecological Adaptations
- The topic of organisms and their environment explores how living beings develop special characteristics to thrive in specific environmental conditions. These adaptations can be morphological, physiological, or behavioral in nature, and they arise through the process of natural selection, either from existing variations or through gradual selection of small adaptive traits.
- The main purpose of these adaptations is to enhance an individual’s ability to obtain food and space for survival, find opportunities for reproduction, and successfully raise offspring.
- Reproduction in humans is a biological process that ensures the continuation of the species. While reproduction is a fundamental aspect of an organism’s life cycle, it is not specifically considered an ecological adaptation. (Hence statement 3 is incorrect)
- Ecological adaptations typically refer to traits or behaviors that directly aid an organism in its interaction with the environment, such as camouflage in animals to blend with their surroundings, drought tolerance in plants to survive arid conditions, or migration in birds to find more favorable habitats during different seasons
Phenotypic Plasticity and Ecotypes
- Phenotype refers to the observable traits resulting from the interaction between an organism’s genetic makeup (genotype) and its surrounding environment. Within a local habitat, variations in environmental conditions lead to diverse phenotypes, a phenomenon known as phenotypic plasticity. (Hence statement 1 is correct)
- These variations give rise to ecotypes, which are genetically adapted local populations of species that exhibit morphological and physiological differences. Despite these genetic variations, ecotypes can interbreed. Ecotypes are commonly observed in plants and sessile animals.
Strategies of Adaptations in Plants
- Plants possess unique characteristics that enable them to tolerate a wide range of environmental conditions, such as light regimes, dryness, high temperatures, water-saturation, and saline environments.
- They have evolved specialized structures in their flowers to ensure pollination by insects or other animals. Additionally, plants have developed various mechanisms to cope with stressful environmental conditions.
Adaptation to Light Regime
Based on their adaptation to different light intensities, plant communities can be categorized into two groups:
- Sciophytes: These are shade-loving plants that exhibit lower levels of photosynthetic, respiratory, and metabolic activities. They typically grow under the dense canopy of forests and include ferns and herbaceous plants. (Hence statement 2 is correct)
- Heliophytes: These plants are adapted to high-intensity light and have higher temperature thresholds for photosynthesis. They also exhibit higher rates of respiration. Examples of heliophytes include shrubs and trees.
-
Question 9 of 35
9. Question
Consider the following statements with reference to the Productivity of biomass:
- Primary productivity refers to the rate of biomass production per unit area by the primary producers
- Gross primary productivity (GPP) is the total amount of energy fixed by the primary producer through photosynthesis
- Net primary productivity (NPP) is the energy remaining after subtracting the energy lost through respiration
- The open ocean contributes the largest percentage of the world’s annual NPP due to its extensive coverage
How many of the above statements are correct?
Correct
Solution (d)
Productivity of biomass
- Productivity is the measure of biomass production per unit area.
- Primary productivity refers to the rate of biomass production per unit area by the primary producers. (Hence statement 1 is correct)
- Primary producers are organisms that produce food through photosynthesis.
- Primary producers occupy the first trophic level in an ecosystem and provide energy to all the consumers.
- Gross primary productivity (GPP) is the total amount of energy fixed by the primary producer through photosynthesis. (Hence statement 2 is correct)
- Net primary productivity (NPP) is the energy remaining after subtracting the energy lost through respiration. (Hence statement 3 is correct)
- NPP represents the actual biomass available for consumption by primary consumers.
- Secondary productivity refers to the rate of new biomass production by primary consumers.
Ecosystem Productivity:
- Tropical rainforests, swamps and marshes, and algae beds and reefs have the highest NPP.
- Open Ocean and desert ecosystems have the lowest NPP.
- The graph illustrates the average NPP (in g/m2/yr) of different ecosystems.
- The third graph shows the world’s annual NPP for each ecosystem type.
- The open ocean contributes the largest percentage of the world’s annual NPP (24.4%) due to its extensive coverage (70% of Earth’s surface). (Hence statement 4 is correct)
- The tropical rainforest has the second highest annual NPP (22%), followed by swamps and marshes (2.3%), and algae beds and reefs (0.9%).
- Therefore, the correct order of ecosystems in increasing NPP is: algae beds and reefs (0.9%), swamps and marshes (2.3%), tropical rainforests (22%), and open oceans (24.4%).
Incorrect
Solution (d)
Productivity of biomass
- Productivity is the measure of biomass production per unit area.
- Primary productivity refers to the rate of biomass production per unit area by the primary producers. (Hence statement 1 is correct)
- Primary producers are organisms that produce food through photosynthesis.
- Primary producers occupy the first trophic level in an ecosystem and provide energy to all the consumers.
- Gross primary productivity (GPP) is the total amount of energy fixed by the primary producer through photosynthesis. (Hence statement 2 is correct)
- Net primary productivity (NPP) is the energy remaining after subtracting the energy lost through respiration. (Hence statement 3 is correct)
- NPP represents the actual biomass available for consumption by primary consumers.
- Secondary productivity refers to the rate of new biomass production by primary consumers.
Ecosystem Productivity:
- Tropical rainforests, swamps and marshes, and algae beds and reefs have the highest NPP.
- Open Ocean and desert ecosystems have the lowest NPP.
- The graph illustrates the average NPP (in g/m2/yr) of different ecosystems.
- The third graph shows the world’s annual NPP for each ecosystem type.
- The open ocean contributes the largest percentage of the world’s annual NPP (24.4%) due to its extensive coverage (70% of Earth’s surface). (Hence statement 4 is correct)
- The tropical rainforest has the second highest annual NPP (22%), followed by swamps and marshes (2.3%), and algae beds and reefs (0.9%).
- Therefore, the correct order of ecosystems in increasing NPP is: algae beds and reefs (0.9%), swamps and marshes (2.3%), tropical rainforests (22%), and open oceans (24.4%).
-
Question 10 of 35
10. Question
Which of the following components is/are considered as biotic components of Environment:
- Topography
- Symbionts
- Geologic substratum
- Non-green plants
- Green plants
Select the correct answer by using the code below:
Correct
Solution (d)
Components of Environment
The environment consists of two fundamental components, which are as follows:
Biotic Components (Biological Component):
- The biotic component encompasses all living organisms within the environment.
- It is also referred to as the biological component of the ecosystem.
- Ecosystems are formed through the interactions between animals, plants, and microorganisms, along with abiotic components.
- The biotic community is categorized into various groups, such as autotrophs, heterotrophs, and saprotrophs.
- These groups occupy different trophic levels in the food chain.
- Autotrophs, also known as producers, form the base of the food chain.
Abiotic Components (Physical Components):
- The physical component refers to the non-living aspects of the environment.
- These components, also known as abiotic variables, include elements such as air, water, soil, and climate.
- They are further classified into the atmosphere, hydrosphere, and lithosphere.
- Scientists also use the term “biosphere” to describe the zone of life, which encompasses the global collection of ecosystems.
Components of Environment Abiotic Biotic · Energy · Radiation
· Temperature & heat flow
· Atmospheric gases and wind
· Water
· Fire
· Gravity
· Topography
· Soil
· Geologic substratum
· Green plants · Non-green plants
· Decomposers
· Parasites
· Symbionts
· Animals
· Man
Incorrect
Solution (d)
Components of Environment
The environment consists of two fundamental components, which are as follows:
Biotic Components (Biological Component):
- The biotic component encompasses all living organisms within the environment.
- It is also referred to as the biological component of the ecosystem.
- Ecosystems are formed through the interactions between animals, plants, and microorganisms, along with abiotic components.
- The biotic community is categorized into various groups, such as autotrophs, heterotrophs, and saprotrophs.
- These groups occupy different trophic levels in the food chain.
- Autotrophs, also known as producers, form the base of the food chain.
Abiotic Components (Physical Components):
- The physical component refers to the non-living aspects of the environment.
- These components, also known as abiotic variables, include elements such as air, water, soil, and climate.
- They are further classified into the atmosphere, hydrosphere, and lithosphere.
- Scientists also use the term “biosphere” to describe the zone of life, which encompasses the global collection of ecosystems.
Components of Environment Abiotic Biotic · Energy · Radiation
· Temperature & heat flow
· Atmospheric gases and wind
· Water
· Fire
· Gravity
· Topography
· Soil
· Geologic substratum
· Green plants · Non-green plants
· Decomposers
· Parasites
· Symbionts
· Animals
· Man
-
Question 11 of 35
11. Question
With reference to the differences between a biome and a biosphere, consider the following statements:
- Biome refers to the living organisms in an ecosystem, while the biosphere refers to the physical environment.
- Biome encompasses a smaller area than the biosphere.
- Biome represents a specific ecological region, while the biosphere encompasses the entire planet.
How many of the above statements are correct?
Correct
Solution (b)
Biome:
- Biomes are large divisions within the terrestrial portion of the biosphere. (Hence statement 2 is correct)
- They are characterized by specific climate patterns, types of vegetation, animal life, and soil characteristics. (not only living organisms) (Hence statement 1 is incorrect)
- Each biome is unique and differs from one another.
- Climate plays a crucial role in determining the boundaries of biomes and the abundance of plant and animal species.
- Temperature and precipitation are the key climatic factors that influence biome characteristics.
- Examples of biomes include tundra, boreal forests, grasslands, deserts, temperate forests, tropical rainforests, tropical deciduous forests, and savannahs.
Biosphere:
- The biosphere refers to the part of the Earth that supports life.
- It encompasses the surface of the Earth, including land, water bodies, and a portion of the atmosphere.
- The biosphere includes all living organisms and their habitats.
- It also encompasses the interaction between organisms and their physical surroundings.
- The biosphere consists of the atmosphere (gaseous envelope surrounding the Earth), lithosphere (solid Earth materials), and hydrosphere (water bodies).
- It represents the interconnected web of life on Earth.
The main difference between a biome and a biosphere is that a biome represents a specific ecological region with distinct characteristics, while the biosphere encompasses the entire planet and encompasses all ecosystems and their interactions. (Hence statement 3 is correct)
Incorrect
Solution (b)
Biome:
- Biomes are large divisions within the terrestrial portion of the biosphere. (Hence statement 2 is correct)
- They are characterized by specific climate patterns, types of vegetation, animal life, and soil characteristics. (not only living organisms) (Hence statement 1 is incorrect)
- Each biome is unique and differs from one another.
- Climate plays a crucial role in determining the boundaries of biomes and the abundance of plant and animal species.
- Temperature and precipitation are the key climatic factors that influence biome characteristics.
- Examples of biomes include tundra, boreal forests, grasslands, deserts, temperate forests, tropical rainforests, tropical deciduous forests, and savannahs.
Biosphere:
- The biosphere refers to the part of the Earth that supports life.
- It encompasses the surface of the Earth, including land, water bodies, and a portion of the atmosphere.
- The biosphere includes all living organisms and their habitats.
- It also encompasses the interaction between organisms and their physical surroundings.
- The biosphere consists of the atmosphere (gaseous envelope surrounding the Earth), lithosphere (solid Earth materials), and hydrosphere (water bodies).
- It represents the interconnected web of life on Earth.
The main difference between a biome and a biosphere is that a biome represents a specific ecological region with distinct characteristics, while the biosphere encompasses the entire planet and encompasses all ecosystems and their interactions. (Hence statement 3 is correct)
-
Question 12 of 35
12. Question
With reference to the Ecotone, consider the following statements:
- A geographical zone where different ecosystems come together and engage with one another
- The presence of edge effect leads to an increase in both species’ diversity and population density within the ecotone
- The species found in the ecotone are identical to those in the adjacent ecosystems.
How many of the above statements are correct?
Correct
Solution (b)
- Ecotone is a zone of junction between two or more diverse ecosystems.
- Examples of ecotones include mangrove forests (between marine and terrestrial ecosystems), grasslands, estuaries, and river banks. (Hence statement 1 is correct)
- It can vary in width, ranging from very narrow to quite wide.
- Ecotone exhibits conditions that are intermediate to the adjacent ecosystems, making it a zone of tension.
- It is linear and shows a progressive increase in species composition of one community while simultaneously decreasing species of the adjoining community.
- Well-developed ecotones may contain organisms that are entirely different from those found in the adjacent communities.
- The edge effect can be observed in ecotones, where the number of species and population density may be greater than in either adjacent community. (Hence statement 2 is correct)
- Organisms primarily or most abundantly found in this zone are known as edge species.
- In terrestrial ecosystems, the edge effect is applicable to birds, with higher bird density observed in the mixed habitat of forest and desert ecotones.
- Ecotones typically have a mixture of species from the adjacent ecosystems rather than an identical species composition. (Hence statement 3 is incorrect)
Incorrect
Solution (b)
- Ecotone is a zone of junction between two or more diverse ecosystems.
- Examples of ecotones include mangrove forests (between marine and terrestrial ecosystems), grasslands, estuaries, and river banks. (Hence statement 1 is correct)
- It can vary in width, ranging from very narrow to quite wide.
- Ecotone exhibits conditions that are intermediate to the adjacent ecosystems, making it a zone of tension.
- It is linear and shows a progressive increase in species composition of one community while simultaneously decreasing species of the adjoining community.
- Well-developed ecotones may contain organisms that are entirely different from those found in the adjacent communities.
- The edge effect can be observed in ecotones, where the number of species and population density may be greater than in either adjacent community. (Hence statement 2 is correct)
- Organisms primarily or most abundantly found in this zone are known as edge species.
- In terrestrial ecosystems, the edge effect is applicable to birds, with higher bird density observed in the mixed habitat of forest and desert ecotones.
- Ecotones typically have a mixture of species from the adjacent ecosystems rather than an identical species composition. (Hence statement 3 is incorrect)
-
Question 13 of 35
13. Question
Consider the following statements with respect to food chain in ecosystem:
- The trophic level at the bottom of the food chain consists of consumers, while higher trophic levels include producers
- Producers within the food chain are capable of generating their own nutrients through the process of photosynthesis
- The food chain depicts the complete exchange of energy and nutrients among organisms within an ecosystem
- A food chain provides alternative food sources to organisms, increasing their chances of survival by offering multiple options for obtaining energy and nutrients
Which of the above given statements is/are incorrect?
Correct
Solution (d)
Food Chain:
- A food chain represents a series of events in an ecosystem where one living organism consumes another organism, which is then consumed by a larger organism.
- The movement of nutrients and energy from one creature to another at different trophic levels is known as a food chain.
- The food chain explains the feeding patterns and relationships between living species.
- Trophic levels refer to the specific order in which producers, primary consumers, secondary consumers, and tertiary consumers appear in a food chain.
- The trophic level at the bottom of the food chain consists of producers, while higher trophic levels include consumers. (Hence statement 1 is incorrect)
Food Chain – Example:
- Grass, using sunlight, produces its own food through photosynthesis.
- The grass is consumed by a grasshopper, which in turn becomes prey for a frog.
- The frog is a food source for a snake, and the snake is hunted by a hawk.
- When a hawk dies, microbes decompose its body, returning nutrients to the soil and benefiting grass-growing plants.
Major Components of Food Chain:
- The Sun: The primary source of energy for all living things and a crucial component of the food chain.
- Producers: Organisms, such as green plants, phytoplankton, and algae that produce their own nutrients through photosynthesis. (Hence statement 2 is correct)
- Consumers: Living species that rely on plants or other organisms for nourishment, including herbivores, carnivores, parasites, and scavengers.
- Decomposers: Organisms that obtain energy from dead organic matter and play a vital role in nutrient recycling.
Types of Food Chains:
- Grazing Food Chain: Begins with green plants, followed by herbivores and carnivores.
- Detritus Food Chain: Starts with decomposing organic matter and involves various organisms such as algae, bacteria, fungi, protozoa, and insects.
Significance of Food Chain:
- Food chains illustrate the complex relationships that exist in an ecosystem.
- They highlight the interdependence of organisms for survival.
- Food chains depict the flow of energy within an ecosystem, showcasing the path of energy movement from producers to consumers.
Food Web
- A food web represents a complex network of interconnected food chains within an ecosystem.
- Unlike a food chain, which depicts a linear pathway of energy flow, a food web shows all possible transfers of energy and nutrients among organisms in an ecosystem. (Hence statement 3 is incorrect)
- In a food web, multiple organisms are involved in consuming and being consumed by others, forming a more realistic representation of energy flow in ecosystems.
- Removing any intermediate component from the food web can have significant impacts on the succeeding links of the chain.
- A food web provides alternative food sources to organisms, increasing their chances of survival by offering multiple options for obtaining energy and nutrients. (Hence statement 4 is incorrect)
- Organisms in a food web can have shifting food preferences and availability, influenced by seasonal changes and other factors.
- The interconnected networks of feeding relationships in a food web ensure the stability and resilience of an ecosystem by providing redundancy and flexibility in energy flow pathways.
Incorrect
Solution (d)
Food Chain:
- A food chain represents a series of events in an ecosystem where one living organism consumes another organism, which is then consumed by a larger organism.
- The movement of nutrients and energy from one creature to another at different trophic levels is known as a food chain.
- The food chain explains the feeding patterns and relationships between living species.
- Trophic levels refer to the specific order in which producers, primary consumers, secondary consumers, and tertiary consumers appear in a food chain.
- The trophic level at the bottom of the food chain consists of producers, while higher trophic levels include consumers. (Hence statement 1 is incorrect)
Food Chain – Example:
- Grass, using sunlight, produces its own food through photosynthesis.
- The grass is consumed by a grasshopper, which in turn becomes prey for a frog.
- The frog is a food source for a snake, and the snake is hunted by a hawk.
- When a hawk dies, microbes decompose its body, returning nutrients to the soil and benefiting grass-growing plants.
Major Components of Food Chain:
- The Sun: The primary source of energy for all living things and a crucial component of the food chain.
- Producers: Organisms, such as green plants, phytoplankton, and algae that produce their own nutrients through photosynthesis. (Hence statement 2 is correct)
- Consumers: Living species that rely on plants or other organisms for nourishment, including herbivores, carnivores, parasites, and scavengers.
- Decomposers: Organisms that obtain energy from dead organic matter and play a vital role in nutrient recycling.
Types of Food Chains:
- Grazing Food Chain: Begins with green plants, followed by herbivores and carnivores.
- Detritus Food Chain: Starts with decomposing organic matter and involves various organisms such as algae, bacteria, fungi, protozoa, and insects.
Significance of Food Chain:
- Food chains illustrate the complex relationships that exist in an ecosystem.
- They highlight the interdependence of organisms for survival.
- Food chains depict the flow of energy within an ecosystem, showcasing the path of energy movement from producers to consumers.
Food Web
- A food web represents a complex network of interconnected food chains within an ecosystem.
- Unlike a food chain, which depicts a linear pathway of energy flow, a food web shows all possible transfers of energy and nutrients among organisms in an ecosystem. (Hence statement 3 is incorrect)
- In a food web, multiple organisms are involved in consuming and being consumed by others, forming a more realistic representation of energy flow in ecosystems.
- Removing any intermediate component from the food web can have significant impacts on the succeeding links of the chain.
- A food web provides alternative food sources to organisms, increasing their chances of survival by offering multiple options for obtaining energy and nutrients. (Hence statement 4 is incorrect)
- Organisms in a food web can have shifting food preferences and availability, influenced by seasonal changes and other factors.
- The interconnected networks of feeding relationships in a food web ensure the stability and resilience of an ecosystem by providing redundancy and flexibility in energy flow pathways.
-
Question 14 of 35
14. Question
How do keystone species differ from foundation species in terms of their impact on community structure?
Correct
Solution (c)
Structure of Community:
- The structure of a community refers to the composition of organisms present in an ecosystem, their abundance, and how they interact with each other.
- Species richness, which is the number of species present, and species diversity, which considers the relative numbers of each species, are used to define the structure of a community.
- Community structure is influenced by various factors, including abiotic influences (such as climate and physical environment), species interactions, disturbance levels, and random events.
- Foundation species and keystone species play crucial roles in shaping the structure of their ecosystems.
- Keystone species are those that have a disproportionately large impact on community structure relative to their biomass or abundance. (Hence option (c) is correct)
- Keystone species are often top predators and exhibit a wider range of behaviors compared to foundation species.
- An example of a keystone species is the intertidal sea star Pisasterochraceus, which regulates the population of mussels in its ecosystem.
- In an experiment where sea stars were removed, the mussel population increased significantly, leading to a decrease in species diversity and the dominance of mussels over other species.
- The loss of a keystone species can result in a decline in diversity or the collapse of community structure, as the interactions that the keystone species maintained are disrupted.
- In the example, the absence of sea stars allowed mussels to outcompete and suppress other species, leading to a loss of diversity in the intertidal zone.
Incorrect
Solution (c)
Structure of Community:
- The structure of a community refers to the composition of organisms present in an ecosystem, their abundance, and how they interact with each other.
- Species richness, which is the number of species present, and species diversity, which considers the relative numbers of each species, are used to define the structure of a community.
- Community structure is influenced by various factors, including abiotic influences (such as climate and physical environment), species interactions, disturbance levels, and random events.
- Foundation species and keystone species play crucial roles in shaping the structure of their ecosystems.
- Keystone species are those that have a disproportionately large impact on community structure relative to their biomass or abundance. (Hence option (c) is correct)
- Keystone species are often top predators and exhibit a wider range of behaviors compared to foundation species.
- An example of a keystone species is the intertidal sea star Pisasterochraceus, which regulates the population of mussels in its ecosystem.
- In an experiment where sea stars were removed, the mussel population increased significantly, leading to a decrease in species diversity and the dominance of mussels over other species.
- The loss of a keystone species can result in a decline in diversity or the collapse of community structure, as the interactions that the keystone species maintained are disrupted.
- In the example, the absence of sea stars allowed mussels to outcompete and suppress other species, leading to a loss of diversity in the intertidal zone.
-
Question 15 of 35
15. Question
With reference to the Pyramid of Biomass, consider the following statements:
- The determination of the pyramid of biomass involves the collection of organisms from individual trophic levels and the subsequent measurement of their dry weight
- The upward pyramid demonstrates that the highest trophic level possesses the greatest amount of biomass
- The base of the Inverted pyramid pyramid consists of primary producers (autotrophs) with the highest biomass
How many of the above statements are correct?
Correct
Solution (a)
Pyramid of Biomass:
- The pyramid of biomass is used to address the limitations of the pyramid of numbers by considering the weight of individuals in each trophic level rather than just counting them.
- The pyramid of biomass is determined by collecting organisms from each trophic level separately and measuring their dry weight. This approach accounts for the size differences among organisms and provides a more accurate representation. (Hence statement 1 is correct)
- Biomass is measured in grams per square meter (g/m²) to quantify the total dry weight of all organisms at each trophic level at a specific time.
(a) Upward pyramid:
- Most land ecosystems exhibit an upward pyramid of biomass.
- The base of the pyramid consists of primary producers (autotrophs) with the highest biomass.
- The trophic levels above the producers show a decreasing biomass, with primary consumers having less biomass than producers, and secondary consumers having less biomass than primary consumers.
- The highest trophic level has the lowest amount of biomass. (Hence statement 2 is incorrect)
(b) Inverted pyramid:
- Many aquatic ecosystems may have an inverted pyramid of biomass.
- In this case, tiny phytoplanktons serve as producers, and they grow and reproduce rapidly.
- The pyramid of biomass in aquatic ecosystems has a small base, with the biomass of consumers at any given moment exceeding the biomass of producers. (Hence statement 3 is incorrect)
- As a result, the pyramid assumes an inverted shape, with higher trophic levels having more biomass than the base.
Incorrect
Solution (a)
Pyramid of Biomass:
- The pyramid of biomass is used to address the limitations of the pyramid of numbers by considering the weight of individuals in each trophic level rather than just counting them.
- The pyramid of biomass is determined by collecting organisms from each trophic level separately and measuring their dry weight. This approach accounts for the size differences among organisms and provides a more accurate representation. (Hence statement 1 is correct)
- Biomass is measured in grams per square meter (g/m²) to quantify the total dry weight of all organisms at each trophic level at a specific time.
(a) Upward pyramid:
- Most land ecosystems exhibit an upward pyramid of biomass.
- The base of the pyramid consists of primary producers (autotrophs) with the highest biomass.
- The trophic levels above the producers show a decreasing biomass, with primary consumers having less biomass than producers, and secondary consumers having less biomass than primary consumers.
- The highest trophic level has the lowest amount of biomass. (Hence statement 2 is incorrect)
(b) Inverted pyramid:
- Many aquatic ecosystems may have an inverted pyramid of biomass.
- In this case, tiny phytoplanktons serve as producers, and they grow and reproduce rapidly.
- The pyramid of biomass in aquatic ecosystems has a small base, with the biomass of consumers at any given moment exceeding the biomass of producers. (Hence statement 3 is incorrect)
- As a result, the pyramid assumes an inverted shape, with higher trophic levels having more biomass than the base.
-
Question 16 of 35
16. Question
Consider the following statements with respect to Pyramid of Energy:
- The energy pyramid is always upward, with a large energy base at the bottom, indicating the availability of more energy at lower trophic levels
- As energy moves from one trophic level to another, there is a decrease in usable energy
- As herbivores consume plants and carnivores consume herbivores, the energy absorption decreases at each step due to utilization for metabolism
How many of the above statements are correct?
Correct
Solution (c)
Pyramid of Energy:
- Purpose: The energy pyramid is used to compare the functional roles of trophic levels in an ecosystem.
- Reflects thermodynamics: The energy pyramid reflects the laws of thermodynamics, showing the conversion of solar energy to chemical energy and heat energy at each trophic level, along with energy loss during transfers.
- Upward orientation: The energy pyramid is always upward, with a large energy base at the bottom, indicating the availability of more energy at lower trophic levels. (Hence statement 1 is correct)
- Energy loss: As energy moves from one trophic level to another, there is a decrease in usable energy. This is due to energy being used for metabolism and lost as heat. (Hence statement 2 is correct)
- Example: Suppose an ecosystem receives 1000 calories of light energy. Only a small portion is absorbed and utilized by green plants, resulting in 100 calories stored as energy-rich materials.
- Energy transfer: When a herbivore (e.g., a deer) consumes the plant, it absorbs a smaller amount of energy (e.g., 10 calories) after using some for its own metabolism. Similarly, a carnivore (e.g., a lion) that consumes the herbivore receives an even smaller amount of energy. (Hence statement 3 is correct)
- Biological magnification: The energy pyramid concept helps explain biological magnification, which is the progressive increase in the concentration of toxic substances at higher levels of the food chain.
Incorrect
Solution (c)
Pyramid of Energy:
- Purpose: The energy pyramid is used to compare the functional roles of trophic levels in an ecosystem.
- Reflects thermodynamics: The energy pyramid reflects the laws of thermodynamics, showing the conversion of solar energy to chemical energy and heat energy at each trophic level, along with energy loss during transfers.
- Upward orientation: The energy pyramid is always upward, with a large energy base at the bottom, indicating the availability of more energy at lower trophic levels. (Hence statement 1 is correct)
- Energy loss: As energy moves from one trophic level to another, there is a decrease in usable energy. This is due to energy being used for metabolism and lost as heat. (Hence statement 2 is correct)
- Example: Suppose an ecosystem receives 1000 calories of light energy. Only a small portion is absorbed and utilized by green plants, resulting in 100 calories stored as energy-rich materials.
- Energy transfer: When a herbivore (e.g., a deer) consumes the plant, it absorbs a smaller amount of energy (e.g., 10 calories) after using some for its own metabolism. Similarly, a carnivore (e.g., a lion) that consumes the herbivore receives an even smaller amount of energy. (Hence statement 3 is correct)
- Biological magnification: The energy pyramid concept helps explain biological magnification, which is the progressive increase in the concentration of toxic substances at higher levels of the food chain.
-
Question 17 of 35
17. Question
Which of the following examples best illustrates the concept of survival of the fittest?
Correct
Solution (d)
Concept of Survival of the Fittest:
- The concept of survival of the fittest stems from Charles Darwin’s theory of natural selection.
- Organisms tend to produce more offspring than can be supported by available resources, leading to competition for survival.
- Genetic variation exists within species, and some variations provide advantages in terms of survival and reproduction.
- Beneficial variations increase an organism’s chances of survival and successful reproduction, leading to the passing on of these advantageous traits to future generations.
- Over time, populations undergo changes as the beneficial traits become more prevalent.
- Survival of the fittest implies that organisms that can adapt to their environment have a higher likelihood of surviving and passing on their advantageous genes.
- Examples of survival of the fittest include the case of moth wing color adaptation during the Industrial Revolution in London and the variation in beak size among Galapagos finches based on environmental conditions.
- The moth population adapted to the changing environment by developing darker wing colors to blend with the soot-covered trees, enhancing their chances of survival.
- Galapagos finches showcase how different beak sizes are favored in different climatic conditions, enabling specific variants to have better survival rates.
- This example demonstrates an adaptation that enhances the lizard’s survival by increasing its chances of avoiding predation. By changing its color to blend with its surroundings, the lizard becomes less visible to potential predators. This adaptation allows the lizard to better survive and pass on its genes to the next generation. (Hence option d) is correct)
- The other examples, while important for the respective organisms, do not directly represent the concept of survival of the fittest in the same way. The bird with strong wings, although beneficial for flying longer distances, does not necessarily imply a higher chance of survival in its environment. The plant producing a large number of seeds is an example of reproductive strategy but does not directly relate to adaptation or survival in a specific environment. The predator capturing and consuming its prey is an illustration of predator-prey interactions but does not solely represent the concept of fitness and adaptation.
Incorrect
Solution (d)
Concept of Survival of the Fittest:
- The concept of survival of the fittest stems from Charles Darwin’s theory of natural selection.
- Organisms tend to produce more offspring than can be supported by available resources, leading to competition for survival.
- Genetic variation exists within species, and some variations provide advantages in terms of survival and reproduction.
- Beneficial variations increase an organism’s chances of survival and successful reproduction, leading to the passing on of these advantageous traits to future generations.
- Over time, populations undergo changes as the beneficial traits become more prevalent.
- Survival of the fittest implies that organisms that can adapt to their environment have a higher likelihood of surviving and passing on their advantageous genes.
- Examples of survival of the fittest include the case of moth wing color adaptation during the Industrial Revolution in London and the variation in beak size among Galapagos finches based on environmental conditions.
- The moth population adapted to the changing environment by developing darker wing colors to blend with the soot-covered trees, enhancing their chances of survival.
- Galapagos finches showcase how different beak sizes are favored in different climatic conditions, enabling specific variants to have better survival rates.
- This example demonstrates an adaptation that enhances the lizard’s survival by increasing its chances of avoiding predation. By changing its color to blend with its surroundings, the lizard becomes less visible to potential predators. This adaptation allows the lizard to better survive and pass on its genes to the next generation. (Hence option d) is correct)
- The other examples, while important for the respective organisms, do not directly represent the concept of survival of the fittest in the same way. The bird with strong wings, although beneficial for flying longer distances, does not necessarily imply a higher chance of survival in its environment. The plant producing a large number of seeds is an example of reproductive strategy but does not directly relate to adaptation or survival in a specific environment. The predator capturing and consuming its prey is an illustration of predator-prey interactions but does not solely represent the concept of fitness and adaptation.
-
Question 18 of 35
18. Question
With reference to the Comparison of Bioaccumulation and Biomagnification, consider the following statements:
- Bioaccumulation is driven by the concentration increase inside an organism, while biomagnification occurs as contaminants travel up the food chain
- Bioaccumulation occurs within a specific trophic level, while biomagnification involves the transfer of pollutants between trophic levels.
Choose the correct code:
Correct
Solution (c)
Bioaccumulation:
- Definition: Bioaccumulation refers to the progressive buildup of contaminants or other compounds in an organism.
- Absorption: Chemicals can enter an organism through breathing, skin absorption, or ingestion.
- Rate of accumulation: Bioaccumulation occurs when the rate of substance accumulation exceeds the rate of removal through catabolism or excretion.
- Persistence: Persistent pollutants like DDT pose a significant risk of bioaccumulation due to their long lifespan, even at low environmental levels.
Biomagnification:
- Definition: Biomagnification is the increasing concentration of a pollutant at each trophic level over time.
- Biological half-life: Biomagnification occurs when the pollutant has a long biological half-life and is soluble in fats rather than water (e.g., DDT).
- Excretion: Water-soluble pollutants are excreted by organisms, while lipid-soluble pollutants remain in the body for longer periods.
- Measurement: Contaminant levels in fatty tissues, such as fish, are commonly measured to assess biomagnification.
- Milk analysis: Milk produced by female mammals, rich in fat, is examined for contaminants due to their potential for biomagnification.
Comparison:
- Scope: Bioaccumulation focuses on the accumulation of substances within an organism, while biomagnification emphasizes the increasing concentration of pollutants across trophic levels.
- Cause: Bioaccumulation is driven by the concentration increase inside an organism, while biomagnification occurs as contaminants travel up the food chain. (Hence statement 1 is correct)
- Level of concentration: Bioaccumulation results in increased concentration within the organism, while biomagnification shows increasing concentration at higher trophic levels.
- Food Chain Interaction: Bioaccumulation occurs within a specific trophic level, while biomagnification involves the transfer of pollutants between trophic levels. (Hence statement 2 is correct)
- Examples: Bioaccumulation is exemplified by mercury accumulation in fish, while biomagnification is illustrated by the transfer of pollutants from microscopic organisms to larger fish and aquatic species.
Incorrect
Solution (c)
Bioaccumulation:
- Definition: Bioaccumulation refers to the progressive buildup of contaminants or other compounds in an organism.
- Absorption: Chemicals can enter an organism through breathing, skin absorption, or ingestion.
- Rate of accumulation: Bioaccumulation occurs when the rate of substance accumulation exceeds the rate of removal through catabolism or excretion.
- Persistence: Persistent pollutants like DDT pose a significant risk of bioaccumulation due to their long lifespan, even at low environmental levels.
Biomagnification:
- Definition: Biomagnification is the increasing concentration of a pollutant at each trophic level over time.
- Biological half-life: Biomagnification occurs when the pollutant has a long biological half-life and is soluble in fats rather than water (e.g., DDT).
- Excretion: Water-soluble pollutants are excreted by organisms, while lipid-soluble pollutants remain in the body for longer periods.
- Measurement: Contaminant levels in fatty tissues, such as fish, are commonly measured to assess biomagnification.
- Milk analysis: Milk produced by female mammals, rich in fat, is examined for contaminants due to their potential for biomagnification.
Comparison:
- Scope: Bioaccumulation focuses on the accumulation of substances within an organism, while biomagnification emphasizes the increasing concentration of pollutants across trophic levels.
- Cause: Bioaccumulation is driven by the concentration increase inside an organism, while biomagnification occurs as contaminants travel up the food chain. (Hence statement 1 is correct)
- Level of concentration: Bioaccumulation results in increased concentration within the organism, while biomagnification shows increasing concentration at higher trophic levels.
- Food Chain Interaction: Bioaccumulation occurs within a specific trophic level, while biomagnification involves the transfer of pollutants between trophic levels. (Hence statement 2 is correct)
- Examples: Bioaccumulation is exemplified by mercury accumulation in fish, while biomagnification is illustrated by the transfer of pollutants from microscopic organisms to larger fish and aquatic species.
-
Question 19 of 35
19. Question
With reference Biotic Interactions, consider the following pairs:
- Parasitism : One species benefits, while the other is harmed
- Commensalism : One species is harmed, while the other is unaffected
- Amensalism : One species is harmed, while the other is unaffected
- Competition : Both species are harmed by the interaction
How many of the above given pairs is/are correctly matched?
Correct
Solution (d)
Types of Biotic Interactions:
- Mutualism: Both species benefit from the interaction.
Example: Pollination mutualisms, where pollinators receive food (pollen, nectar) while plants achieve cross-fertilization.
- Commensalism: One species benefit, while the other is unaffected. Example: Dung beetles benefit from cow dung, which provides them with food and shelter, while the cows are not affected. (Hence pair 2 is correctly matched)
- Competition: Both species are harmed by the interaction. (Hence pair 4 is correctly matched)
Example: When two species compete for the same food and there is insufficient supply, both species suffer from a shortage of food.
- Predation and Parasitism: One species benefit, while the other is harmed. (Hence pair 1 is correctly matched)
Example: Predation occurs when one fish kills and eats another, while parasitism is seen in ticks gaining benefits by sucking blood, which harms the host.
- Amensalism: One species is harmed, while the other is unaffected. Example: A large tree casting shade on a small plant, inhibiting its growth, while the small plant has no effect on the tree. (Hence pair 3 is correctly matched)
- Neutralism: No net benefit or harm to either species. Example: In some interactions, the costs and benefits experienced by each partner are exactly the same, resulting in a neutral relationship. It can also refer to species inhabiting the same space and utilizing the same resources without influencing each other significantly.
Incorrect
Solution (d)
Types of Biotic Interactions:
- Mutualism: Both species benefit from the interaction.
Example: Pollination mutualisms, where pollinators receive food (pollen, nectar) while plants achieve cross-fertilization.
- Commensalism: One species benefit, while the other is unaffected. Example: Dung beetles benefit from cow dung, which provides them with food and shelter, while the cows are not affected. (Hence pair 2 is correctly matched)
- Competition: Both species are harmed by the interaction. (Hence pair 4 is correctly matched)
Example: When two species compete for the same food and there is insufficient supply, both species suffer from a shortage of food.
- Predation and Parasitism: One species benefit, while the other is harmed. (Hence pair 1 is correctly matched)
Example: Predation occurs when one fish kills and eats another, while parasitism is seen in ticks gaining benefits by sucking blood, which harms the host.
- Amensalism: One species is harmed, while the other is unaffected. Example: A large tree casting shade on a small plant, inhibiting its growth, while the small plant has no effect on the tree. (Hence pair 3 is correctly matched)
- Neutralism: No net benefit or harm to either species. Example: In some interactions, the costs and benefits experienced by each partner are exactly the same, resulting in a neutral relationship. It can also refer to species inhabiting the same space and utilizing the same resources without influencing each other significantly.
-
Question 20 of 35
20. Question
With reference to the nitrogen cycle, consider the following statements:
- The nitrogen cycle starts with nitrogen fixation, converting atmospheric nitrogen into ammonia through symbiotic bacteria like Azotobacter and Rhizobium
- Ammonification converts organic nitrogen from dead plants or animals into ammonium through the action of decomposers like bacteria and fungi
- Denitrification is the final stage of the nitrogen cycle, where bacteria convert nitrates into gaseous nitrogen in oxygen-depleted environments, releasing nitrogen compounds into the atmosphere
How many of the above statements are correct?
Correct
Solution (c)
Nitrogen Cycle
- The nitrogen cycle is a biogeochemical process that converts inert nitrogen in the atmosphere into a form that can be used by living organisms. It is a vital component of the ecosystem.
- The nitrogen cycle involves several processes, including nitrogen fixation, nitrification, assimilation, ammonification, and denitrification.
- Nitrogen exists in both organic and inorganic forms. Organic nitrogen is found in living organisms and is transferred through the food chain. Inorganic nitrogen is abundant in the atmosphere.
- Nitrogen-fixing bacteria play a crucial role in converting atmospheric nitrogen into nitrogen compounds, such as nitrates and nitrites, which can be utilized by plants.
- The nitrogen cycle begins with nitrogen fixation, where atmospheric nitrogen is transformed into ammonia by symbiotic bacteria. This process is facilitated by diazotrophs like Azotobacter and Rhizobium. (Hence statement 1 is correct)
- Nitrification is the process in which ammonia is converted into nitrate by microorganisms in the soil. This involves the oxidation of ammonia to nitrite, followed by the conversion of nitrite to nitrate.
- Assimilation occurs when plants absorb nitrogen compounds from the soil through their roots. These compounds are used in the production of plant and animal proteins, entering the food web.
- Ammonification is the process of converting organic nitrogen from dead plants or animals into ammonium. Decomposers, such as bacteria and fungi, break down organic matter and release ammonia. (Hence statement 2 is correct)
- Denitrification is the final stage of the nitrogen cycle, where bacteria convert nitrates back into gaseous nitrogen, releasing nitrogen compounds into the atmosphere. This process occurs in oxygen-depleted environments. (Hence statement 3 is correct)
- In the marine ecosystem, the nitrogen cycle is similar to the terrestrial ecosystem. Marine microorganisms, including cyanobacteria and phytoplankton, play a crucial role in nitrogen fixation, nitrification, and other nitrogen cycle processes.
- The nitrogen cycle helps maintain ecosystem balance and nutrient availability for plants and animals. It is essential for the growth and survival of organisms in both terrestrial and marine environments.
Incorrect
Solution (c)
Nitrogen Cycle
- The nitrogen cycle is a biogeochemical process that converts inert nitrogen in the atmosphere into a form that can be used by living organisms. It is a vital component of the ecosystem.
- The nitrogen cycle involves several processes, including nitrogen fixation, nitrification, assimilation, ammonification, and denitrification.
- Nitrogen exists in both organic and inorganic forms. Organic nitrogen is found in living organisms and is transferred through the food chain. Inorganic nitrogen is abundant in the atmosphere.
- Nitrogen-fixing bacteria play a crucial role in converting atmospheric nitrogen into nitrogen compounds, such as nitrates and nitrites, which can be utilized by plants.
- The nitrogen cycle begins with nitrogen fixation, where atmospheric nitrogen is transformed into ammonia by symbiotic bacteria. This process is facilitated by diazotrophs like Azotobacter and Rhizobium. (Hence statement 1 is correct)
- Nitrification is the process in which ammonia is converted into nitrate by microorganisms in the soil. This involves the oxidation of ammonia to nitrite, followed by the conversion of nitrite to nitrate.
- Assimilation occurs when plants absorb nitrogen compounds from the soil through their roots. These compounds are used in the production of plant and animal proteins, entering the food web.
- Ammonification is the process of converting organic nitrogen from dead plants or animals into ammonium. Decomposers, such as bacteria and fungi, break down organic matter and release ammonia. (Hence statement 2 is correct)
- Denitrification is the final stage of the nitrogen cycle, where bacteria convert nitrates back into gaseous nitrogen, releasing nitrogen compounds into the atmosphere. This process occurs in oxygen-depleted environments. (Hence statement 3 is correct)
- In the marine ecosystem, the nitrogen cycle is similar to the terrestrial ecosystem. Marine microorganisms, including cyanobacteria and phytoplankton, play a crucial role in nitrogen fixation, nitrification, and other nitrogen cycle processes.
- The nitrogen cycle helps maintain ecosystem balance and nutrient availability for plants and animals. It is essential for the growth and survival of organisms in both terrestrial and marine environments.
-
Question 21 of 35
21. Question
Consider the following statements:
- World Hepatitis Day is observed each year on the birthday of Dr. Baruch Blumberg.
- The theme for World Hepatitis Day 2023 is “We’re not waiting”.
- Hepatitis is the inflammation of the liver, characterised by irritation or swelling of the liver cells due to various causes.
How many of the above statements are correct?
Correct
Solution (c)
- World Hepatitis Day is observed each year on the birthday of Dr. Baruch Blumberg. Hence statement 1 is correct.
- Blumberg discovered the hepatitis B virus in 1967,and 2 years later, he developed the first hepatitis B vaccine.
- These achievements culminated in Dr. Blumberg winning the Nobel Prize in Physiology or Medicine in 1976.
- World Hepatitis Day is observed each year on July 28 to raise awareness of viral hepatitis, which causes inflammation of the liverthat leads to severe disease and liver cancer.
- It is recognized by the World Health Organization (WHO).
- The theme for World Hepatitis Day 2023 is “We’re not waiting”. Hence statement 2 is correct.
- This theme highlights the need for urgent action to eliminate viral hepatitis by 2030.
- Hepatitis is the inflammation of the liver, characterised by irritation or swelling of the liver cells due to various causes. Hence statement 3 is correct.
- Hepatitis can be an acute (short-term) infection or a chronic (long-term) infection.
- There aredifferent types of hepatitis, with different causes:
- Viral hepatitis is the most common type. It is caused by one of several viruses — hepatitis viruses A, B, C, D, and E.
- Alcoholic hepatitisis caused by heavy alcohol use.
- Toxic hepatitis can be caused by certain poisons, chemicals, medicines, or supplements.
- Autoimmune hepatitis is a chronic type in which your body’s immune system attacks your liver.The cause is not known, but genetics and your environment may play a role.
- Some people with hepatitis do not have symptoms and do not know they are infected. Common symptoms include:
- Fever
- Fatigue
- Loss of appetite
- Nausea and/or vomiting
- Abdominal pain
- Dark urine
- Clay-colored bowel movements
- Joint pain
- Jaundice, yellowing of your skin and eyes
- Treatment for hepatitis depends on which type you have and whether it is acute or chronic.
- Acute viral hepatitis often goes away on its own. There are different medicinesto treat the different chronic types of hepatitis.
Incorrect
Solution (c)
- World Hepatitis Day is observed each year on the birthday of Dr. Baruch Blumberg. Hence statement 1 is correct.
- Blumberg discovered the hepatitis B virus in 1967,and 2 years later, he developed the first hepatitis B vaccine.
- These achievements culminated in Dr. Blumberg winning the Nobel Prize in Physiology or Medicine in 1976.
- World Hepatitis Day is observed each year on July 28 to raise awareness of viral hepatitis, which causes inflammation of the liverthat leads to severe disease and liver cancer.
- It is recognized by the World Health Organization (WHO).
- The theme for World Hepatitis Day 2023 is “We’re not waiting”. Hence statement 2 is correct.
- This theme highlights the need for urgent action to eliminate viral hepatitis by 2030.
- Hepatitis is the inflammation of the liver, characterised by irritation or swelling of the liver cells due to various causes. Hence statement 3 is correct.
- Hepatitis can be an acute (short-term) infection or a chronic (long-term) infection.
- There aredifferent types of hepatitis, with different causes:
- Viral hepatitis is the most common type. It is caused by one of several viruses — hepatitis viruses A, B, C, D, and E.
- Alcoholic hepatitisis caused by heavy alcohol use.
- Toxic hepatitis can be caused by certain poisons, chemicals, medicines, or supplements.
- Autoimmune hepatitis is a chronic type in which your body’s immune system attacks your liver.The cause is not known, but genetics and your environment may play a role.
- Some people with hepatitis do not have symptoms and do not know they are infected. Common symptoms include:
- Fever
- Fatigue
- Loss of appetite
- Nausea and/or vomiting
- Abdominal pain
- Dark urine
- Clay-colored bowel movements
- Joint pain
- Jaundice, yellowing of your skin and eyes
- Treatment for hepatitis depends on which type you have and whether it is acute or chronic.
- Acute viral hepatitis often goes away on its own. There are different medicinesto treat the different chronic types of hepatitis.
-
Question 22 of 35
22. Question
Consider the following statements regarding the climatic conditions required for Cocoa:
- It requires an annual rainfall of 300-500 cm.
- It grows in the temperature range of 15°-39°C with an optimum of 25°C considered ideal.
- The majority of the area under Cocoa cultivation is on clay loam and sandy loam soil.
How many of the above statements are correct?
Correct
Solution (b)
- Cocoa is an important plantation crop grown for chocolates around the world.
- It is known as a crop of humid tropics and is native to the Amazon basin of South America.
- It is mainly grown in an area of land around the equator between 20 degrees latitude north and south.
- About 70 percent of the world’s cocoa beans come from four West African countries – Ivory Coast, Ghana, Nigeria, and Cameroon.
The climatic conditions required for Cocoa:
- It requires an annual rainfall of 150-200 cm. Hence statement 1 is incorrect.
- It grows in the temperature range of 15°-39°C with an optimum of 25°C considered ideal. Hence statement 2 is correct.
- The majority of the area under Cocoa cultivation is on clay loam and sandy loam soil. It grows well in the pH range of 6.5 to 7.0. Hence statement 3 is correct.
Incorrect
Solution (b)
- Cocoa is an important plantation crop grown for chocolates around the world.
- It is known as a crop of humid tropics and is native to the Amazon basin of South America.
- It is mainly grown in an area of land around the equator between 20 degrees latitude north and south.
- About 70 percent of the world’s cocoa beans come from four West African countries – Ivory Coast, Ghana, Nigeria, and Cameroon.
The climatic conditions required for Cocoa:
- It requires an annual rainfall of 150-200 cm. Hence statement 1 is incorrect.
- It grows in the temperature range of 15°-39°C with an optimum of 25°C considered ideal. Hence statement 2 is correct.
- The majority of the area under Cocoa cultivation is on clay loam and sandy loam soil. It grows well in the pH range of 6.5 to 7.0. Hence statement 3 is correct.
-
Question 23 of 35
23. Question
Consider the following statements regarding the Bugun Tribe:
- They mainly reside adjacent to Dibru-Saikhowa National Park in Assam.
- They celebrate the annual festival called the “Pham-Kho”.
- They have their own socio-political administrative decision-making body called the Nimiang.
How many of the above statements are correct?
Correct
Solution (b)
- The Bugun Tribe mainly resides adjacent to Eaglenest Wildlife Sanctuary in Arunachal Pradesh. Hence statement 1 is incorrect.
- They migrated from Tibet via East Kameng, Arunachal Pradesh.
- They have their own geographical boundaries with neighbouring tribes and have a distinct language called Khowa.
- In ancient times the Bugun/Khowa tribe was known as “Shisung-Rubai.”
- They celebrate the annual festival called the “Pham-Kho”. Hence statement 2 is correct.
- Like other Indo-Mongoloid tribes of Arunachal Pradesh, Buguns (Khowas) too build their houses with bamboo.
- They are skilled at extracting oil from a local wood species found in the forests called Chiblem.
- They are generally endogamous as they marry within their community.
- They have their own socio-political administrative decision-making body called the Nimiang. Hence statement 3 is correct.
Incorrect
Solution (b)
- The Bugun Tribe mainly resides adjacent to Eaglenest Wildlife Sanctuary in Arunachal Pradesh. Hence statement 1 is incorrect.
- They migrated from Tibet via East Kameng, Arunachal Pradesh.
- They have their own geographical boundaries with neighbouring tribes and have a distinct language called Khowa.
- In ancient times the Bugun/Khowa tribe was known as “Shisung-Rubai.”
- They celebrate the annual festival called the “Pham-Kho”. Hence statement 2 is correct.
- Like other Indo-Mongoloid tribes of Arunachal Pradesh, Buguns (Khowas) too build their houses with bamboo.
- They are skilled at extracting oil from a local wood species found in the forests called Chiblem.
- They are generally endogamous as they marry within their community.
- They have their own socio-political administrative decision-making body called the Nimiang. Hence statement 3 is correct.
-
Question 24 of 35
24. Question
Consider the following statements regarding Asbestos:
- They are a group of six silicate minerals with similar but distinct properties.
- They are non-flammable even at very high temperatures, and resistant to heat and corrosion.
- India’s asbestos requirement is met through imports from Russia, Kazakhstan, Brazil, and China.
How many of the above statements are correct?
Correct
Solution (c)
- Asbestos is a group of six silicate minerals with similar but distinct properties. Hence statement 1 is correct.
- These are generally divided into two sub-groups – serpentine and amphiboles. Serpentine asbestos (chrysotile or white asbestos) was the most commonly used type of asbestos.
- They are non-flammable even at very high temperatures, and resistant to heat and corrosion. Hence statement 2 is correct.
- They are extremely flexible and durable.
- They have good tensile strength and have low heat conductivity and high resistance to electricity.
- India’s asbestos requirement is met through imports from Russia, Kazakhstan, Brazil, and China. Hence statement 3 is correct.
- According to the World Health Organization, all varieties of asbestos are associated with conditions such as lung cancer, mesothelioma, laryngeal cancer, ovarian cancer, and asbestosis, lung fibrosis.
- If products containing asbestos are disturbed, tiny asbestos fibers are released into the air.
- When asbestos fibers are breathed in, they may get trapped in the lungs and remain there for a long time.
Incorrect
Solution (c)
- Asbestos is a group of six silicate minerals with similar but distinct properties. Hence statement 1 is correct.
- These are generally divided into two sub-groups – serpentine and amphiboles. Serpentine asbestos (chrysotile or white asbestos) was the most commonly used type of asbestos.
- They are non-flammable even at very high temperatures, and resistant to heat and corrosion. Hence statement 2 is correct.
- They are extremely flexible and durable.
- They have good tensile strength and have low heat conductivity and high resistance to electricity.
- India’s asbestos requirement is met through imports from Russia, Kazakhstan, Brazil, and China. Hence statement 3 is correct.
- According to the World Health Organization, all varieties of asbestos are associated with conditions such as lung cancer, mesothelioma, laryngeal cancer, ovarian cancer, and asbestosis, lung fibrosis.
- If products containing asbestos are disturbed, tiny asbestos fibers are released into the air.
- When asbestos fibers are breathed in, they may get trapped in the lungs and remain there for a long time.
-
Question 25 of 35
25. Question
Consider the following statements regarding the International Seabed Authority (ISA):
- It is an autonomous international organization established to regulate mining and related activities in the international seabed beyond national jurisdiction.
- It came into existence upon the entry into force of the United Nations Convention on the Law of the Sea (UNCLOS).
- It is headquartered in Lyon, France.
How many of the above statements are correct?
Correct
Solution (b)
- The International Seabed Authority (ISA) is an autonomous international organization established to regulate mining and related activities in the international seabed beyond national jurisdiction. Hence statement 1 is correct.
- It is the organization through which state parties to UNCLOS organize and control all mineral-resources-related activities in the Area (the seabed and ocean floor and the subsoil thereof, beyond the limits of national jurisdiction) for the benefit of humankind as a whole.
- In so doing, ISA has the mandate to ensure the effective protection of the marine environment from harmful effects that may arise from deep-seabed-related activities.
- It came into existence upon the entry into force of the United Nations Convention on the Law of the Sea (UNCLOS). Hence statement 2 is correct.
- UNCLOS codified international law regarding territorial waters, sea lanes, and ocean resources.
- ISA has 169 Members, including 168 Member States and the European Union.
- It is headquartered in Kingston, Jamaica. Hence statement 3 is incorrect.
Incorrect
Solution (b)
- The International Seabed Authority (ISA) is an autonomous international organization established to regulate mining and related activities in the international seabed beyond national jurisdiction. Hence statement 1 is correct.
- It is the organization through which state parties to UNCLOS organize and control all mineral-resources-related activities in the Area (the seabed and ocean floor and the subsoil thereof, beyond the limits of national jurisdiction) for the benefit of humankind as a whole.
- In so doing, ISA has the mandate to ensure the effective protection of the marine environment from harmful effects that may arise from deep-seabed-related activities.
- It came into existence upon the entry into force of the United Nations Convention on the Law of the Sea (UNCLOS). Hence statement 2 is correct.
- UNCLOS codified international law regarding territorial waters, sea lanes, and ocean resources.
- ISA has 169 Members, including 168 Member States and the European Union.
- It is headquartered in Kingston, Jamaica. Hence statement 3 is incorrect.
-
Question 26 of 35
26. Question
Consider the following statements regarding the Netravati River:
- It rises between Kudrermukh and Ballalaryan Durga in the Dakshina Kannada district of Karnataka.
- The Kumaradhara and Hemavati are its tributaries.
Choose the correct code:
Correct
Solution (c)
- The Netravati River rises between Kudrermukh and Ballalaryan Durga in the Dakshina Kannada district of Karnataka. Hence statement 1 is correct.
- The river drains an area of 3,657 sq km.
- During the monsoon season, the Netravati River overflows, which has an adverse effect on the nearby areas.
- Under the world’s biodiversity conservation project, the catchment area of the Netravati River is selected as one of the 30 hotspots for biodiversity conservation.
- It flows generally in a North-South direction for 40 km up to Gohattu, where it takes a turn towards the West and thereafter flows in an East-West direction up to its outfall into the Arabian Sea near Mangalore.
- The Kumaradhara and Hemavati are its tributaries. Hence statement 2 is correct.
Incorrect
Solution (c)
- The Netravati River rises between Kudrermukh and Ballalaryan Durga in the Dakshina Kannada district of Karnataka. Hence statement 1 is correct.
- The river drains an area of 3,657 sq km.
- During the monsoon season, the Netravati River overflows, which has an adverse effect on the nearby areas.
- Under the world’s biodiversity conservation project, the catchment area of the Netravati River is selected as one of the 30 hotspots for biodiversity conservation.
- It flows generally in a North-South direction for 40 km up to Gohattu, where it takes a turn towards the West and thereafter flows in an East-West direction up to its outfall into the Arabian Sea near Mangalore.
- The Kumaradhara and Hemavati are its tributaries. Hence statement 2 is correct.
-
Question 27 of 35
27. Question
Consider the following statements about the Earth Hour:
- It is a global grassroots movement uniting people to take action on environmental issues and protect the planet.
- It is organized by the International Energy Agency (IEA).
- It takes place towards the end of March every year.
How many of the above statements are correct?
Correct
Solution (b)
- The Earth Hour is a global grassroots movement uniting people to take action on environmental issues and protect the planet. Hence statement 1 is correct.
- It started in Sydney, Australia, in 2007 as a symbolic lights-out event and has since grown into a global movement involving millions of people in over 190 countries and territories.
- It is organized by the World Wildlife Fund (WWF). Hence statement 2 is incorrect.
- It encourages people to switch off all lights for an hour, from 8:30 p.m. to 9:30 p.m. local time, to promote awareness of climate change challenges and energy conservation.
- Governments and companies also participate by turning off non-essential lights in their buildings, monuments, and landmarks to raise awareness about the impact of energy consumption on our planet.
- It takes place towards the end of March every year. Hence statement 3 is correct.
- It is known as the ‘lights off’ moment, unites people worldwide in a show of support for the planet and serves as a reminder of the environmental issues facing us.
Incorrect
Solution (b)
- The Earth Hour is a global grassroots movement uniting people to take action on environmental issues and protect the planet. Hence statement 1 is correct.
- It started in Sydney, Australia, in 2007 as a symbolic lights-out event and has since grown into a global movement involving millions of people in over 190 countries and territories.
- It is organized by the World Wildlife Fund (WWF). Hence statement 2 is incorrect.
- It encourages people to switch off all lights for an hour, from 8:30 p.m. to 9:30 p.m. local time, to promote awareness of climate change challenges and energy conservation.
- Governments and companies also participate by turning off non-essential lights in their buildings, monuments, and landmarks to raise awareness about the impact of energy consumption on our planet.
- It takes place towards the end of March every year. Hence statement 3 is correct.
- It is known as the ‘lights off’ moment, unites people worldwide in a show of support for the planet and serves as a reminder of the environmental issues facing us.
-
Question 28 of 35
28. Question
Consider the following statements regarding the Battle of Okinawa:
- It was the last major battle of World War I.
- It was fought between the U.S. and Germany.
- Okinawa is the largest of the Ryukyu Islands.
How many of the above statements are correct?
Correct
Solution (a)
- The Battle of Okinawa was the last major battle of World War II. Hence statement 1 is incorrect.
- The Americans wanted control of Okinawa because it had four airfields and could support tactical and strategic air operations.
- Capturing Okinawa was regarded as a vital precursor to a ground invasion of the Japanese home islands.
- Code named Operation Iceberg, the invasion of Okinawa and other islands in the Ryukyus began on April 1, 1945.
- It was fought between the U.S. and Japan. Hence statement 2 is incorrect.
- The immense size of the invasion forces made it the largest amphibious assault in the Pacific War.
- Okinawa is the largest of the Ryukyu Islands. Hence statement 3 is correct.
- It lies 350 miles from mainland Japan.
- Ryukyus Islands is an archipelago, extending some 700 miles (1,100 km) south-westward from the southern Japanese island of Kyushu to north-eastern Taiwan.
Incorrect
Solution (a)
- The Battle of Okinawa was the last major battle of World War II. Hence statement 1 is incorrect.
- The Americans wanted control of Okinawa because it had four airfields and could support tactical and strategic air operations.
- Capturing Okinawa was regarded as a vital precursor to a ground invasion of the Japanese home islands.
- Code named Operation Iceberg, the invasion of Okinawa and other islands in the Ryukyus began on April 1, 1945.
- It was fought between the U.S. and Japan. Hence statement 2 is incorrect.
- The immense size of the invasion forces made it the largest amphibious assault in the Pacific War.
- Okinawa is the largest of the Ryukyu Islands. Hence statement 3 is correct.
- It lies 350 miles from mainland Japan.
- Ryukyus Islands is an archipelago, extending some 700 miles (1,100 km) south-westward from the southern Japanese island of Kyushu to north-eastern Taiwan.
-
Question 29 of 35
29. Question
Consider the following statements regarding Biomining:
- It is a method of extracting metals from ores and solid materials using prokaryotes, fungi, or plants.
- It targets various metals, including copper, gold, uranium, and rare earth elements.
- It reduces energy consumption, water usage, and carbon emissions compared to traditional mining methods.
How many of the above statements are correct?
Correct
Solution (c)
- Biomining is a method of extracting metals from ores and solid materials using prokaryotes, fungi, or plants. Hence statement 1 is correct.
- Microorganisms such as bacteria and archaea secrete organic compounds that chelate metals from the environment, making them accessible for extraction.
- It may also be used to clean up sites that have been polluted with metals.
- It targets various metals, including copper, gold, uranium, and rare earth elements. Hence statement 2 is correct.
- It primarily involves bio-oxidation and bioleaching processes.
- Large-scale biomining operations utilize chemostats of microbes to leach metals from ores, offering a sustainable and eco-friendly alternative to traditional mining methods.
- It reduces energy consumption, water usage, and carbon emissions compared to traditional mining methods. Hence statement 3 is correct.
- It permits extraction from low-grade ores, contributing to resource recovery and waste remediation.
Incorrect
Solution (c)
- Biomining is a method of extracting metals from ores and solid materials using prokaryotes, fungi, or plants. Hence statement 1 is correct.
- Microorganisms such as bacteria and archaea secrete organic compounds that chelate metals from the environment, making them accessible for extraction.
- It may also be used to clean up sites that have been polluted with metals.
- It targets various metals, including copper, gold, uranium, and rare earth elements. Hence statement 2 is correct.
- It primarily involves bio-oxidation and bioleaching processes.
- Large-scale biomining operations utilize chemostats of microbes to leach metals from ores, offering a sustainable and eco-friendly alternative to traditional mining methods.
- It reduces energy consumption, water usage, and carbon emissions compared to traditional mining methods. Hence statement 3 is correct.
- It permits extraction from low-grade ores, contributing to resource recovery and waste remediation.
-
Question 30 of 35
30. Question
Consider the following statements regarding Reverse Flipping:
- It is used to describe the trend of overseas start-ups shifting their domicile to India and listing on Indian stock exchanges.
- The process such as simplifying the processes for tax vacations, capital movements, and decreasing tax layers can accelerate it.
Choose the correct code:
Correct
Solution (c)
- Reverse Flipping is used to describe the trend of overseas start-ups shifting their domicile to India and listing on Indian stock exchanges. Hence statement 1 is correct.
- The general motivation for a reverse flip is the increased certainty of an exit at a higher valuation in India.
- This trend has been gaining traction in recent years, as start-ups look to capitalise on India’s large and growing economy, access to deeper pools of venture capital, favourable tax regimes, better intellectual property protection, a young and educated population, and favourable government policies.
- The process such as simplifying the processes for tax vacations, capital movements, and decreasing tax layers can accelerate it. These are proposed in the Economic Survey 2022-23. Hence statement 2 is correct.
Incorrect
Solution (c)
- Reverse Flipping is used to describe the trend of overseas start-ups shifting their domicile to India and listing on Indian stock exchanges. Hence statement 1 is correct.
- The general motivation for a reverse flip is the increased certainty of an exit at a higher valuation in India.
- This trend has been gaining traction in recent years, as start-ups look to capitalise on India’s large and growing economy, access to deeper pools of venture capital, favourable tax regimes, better intellectual property protection, a young and educated population, and favourable government policies.
- The process such as simplifying the processes for tax vacations, capital movements, and decreasing tax layers can accelerate it. These are proposed in the Economic Survey 2022-23. Hence statement 2 is correct.
-
Question 31 of 35
31. Question
From a point P on a level ground, the angle of elevation of the top tower is 300. If the tower is 100 m high, the distance of point P from the foot of the tower is:
Correct
Solution (c)
Let AB be the tower.
Then, ∠APB = 30° and AB = 100m
AB/AP = tan 30° = 1/√3
⇒ AP = AB × √3 = 100 × √3
⇒ AP = 100 × 1.73 = 173m
Incorrect
Solution (c)
Let AB be the tower.
Then, ∠APB = 30° and AB = 100m
AB/AP = tan 30° = 1/√3
⇒ AP = AB × √3 = 100 × √3
⇒ AP = 100 × 1.73 = 173m
-
Question 32 of 35
32. Question
Let a, b, c and d be natural numbers such that
a – 502 = b + 503 = c – 504 = d + 506
Which one of the following is the largest natural number?
Correct
Solution (c)
In questions like this, we need to keep in mind that we are just supposed to compare the numbers and so
we should avoid wasting our time in finding their real values.
a – 502 = b + 503
or a = b + 503 + 502 (it means a > b)
b + 503 = c – 504
or c = b + 503 + 504 (it means c > b)
c – 504 = d + 505
or c = d + 505 + 504 (it means c > d)
a – 502 = c – 504
or c = a – 502 + 504 = a + 2 (it means c > a)
We already know that c > b, d
On arranging all letters according to above findings, we get:
c > a > b
& c > d
Hence, c is the largest number.
Incorrect
Solution (c)
In questions like this, we need to keep in mind that we are just supposed to compare the numbers and so
we should avoid wasting our time in finding their real values.
a – 502 = b + 503
or a = b + 503 + 502 (it means a > b)
b + 503 = c – 504
or c = b + 503 + 504 (it means c > b)
c – 504 = d + 505
or c = d + 505 + 504 (it means c > d)
a – 502 = c – 504
or c = a – 502 + 504 = a + 2 (it means c > a)
We already know that c > b, d
On arranging all letters according to above findings, we get:
c > a > b
& c > d
Hence, c is the largest number.
-
Question 33 of 35
33. Question
x is a prime number. It is squared, and added to a different prime number.
Statements:
- The resultant is a positive number.
- The resultant is an odd number.
- The resultant is an even number.
- The resultant is a negative number.
Based on the above statements chose the correct option
Correct
Solution (d)
Case 1:
Let x = 2
Required sum = 22+ 3 = 4 + 3 = 7
So, the resultant is odd. So, statement ii is correct.
Case 2:
Let x = 3
Required sum = 32 + 5 = 9 + 5 = 14
So, the resultant is even. So, statement iii is correct.
Both the results are positive numbers. So statement i is also true.
So, option (d) is the correct answer.
Incorrect
Solution (d)
Case 1:
Let x = 2
Required sum = 22+ 3 = 4 + 3 = 7
So, the resultant is odd. So, statement ii is correct.
Case 2:
Let x = 3
Required sum = 32 + 5 = 9 + 5 = 14
So, the resultant is even. So, statement iii is correct.
Both the results are positive numbers. So statement i is also true.
So, option (d) is the correct answer.
-
Question 34 of 35
34. Question
In the repeating decimal below, the digits 9639757 repeat. What digit is in the 768th place to the right of the decimal point?
0.96397579639757…
Correct
Solution (d)
There is 7 digits in the given number, so it will repeat after every 7 digit and 7th digit of the number is 7.
It will look like:
1st 2nd 3rd 4th 5th 6th 7th 8th 9th 10th 11th 12th 13th 14th
9 6 3 9 7 5 7 9 6 3 9 7 5 7
763rd 764th 765th 766th 767th 768th
7 9 6 3 9 7
763 is the closest number that is a multiple of 7. Hence, 768th place will be 7.
So, option (d) is the correct option.
Incorrect
Solution (d)
There is 7 digits in the given number, so it will repeat after every 7 digit and 7th digit of the number is 7.
It will look like:
1st 2nd 3rd 4th 5th 6th 7th 8th 9th 10th 11th 12th 13th 14th
9 6 3 9 7 5 7 9 6 3 9 7 5 7
763rd 764th 765th 766th 767th 768th
7 9 6 3 9 7
763 is the closest number that is a multiple of 7. Hence, 768th place will be 7.
So, option (d) is the correct option.
-
Question 35 of 35
35. Question
A monkey tries to pick a coconut from a coconut tree which is at the height of 61 feet. Every time the monkey jumps 3 feet and slides down 1 feet. What is the number of jumps required for the monkey to reach to the coconut?
Correct
Solution (c)
Height of coconut = 61 feet
Total height covered by monkey in one jump = 3 – 1 = 2 feet
Number of jumps required by monkey to cover 58 feet = 58/2 = 29 jumps
In the 30th jump the monkey will reach the coconut.
Hence, total number of jumps required = 30
Hence, the correct answer is option (c).
Incorrect
Solution (c)
Height of coconut = 61 feet
Total height covered by monkey in one jump = 3 – 1 = 2 feet
Number of jumps required by monkey to cover 58 feet = 58/2 = 29 jumps
In the 30th jump the monkey will reach the coconut.
Hence, total number of jumps required = 30
Hence, the correct answer is option (c).
All the Best
IASbaba